Which one of the following, if substituted for the condition that Lewis and Ota do not lecture on the same day as eac...

sprozes on January 6, 2020

More detailed explanation please

Could someone please explain this in a more detailed manner. Thank you.

Reply
Create a free account to read and take part in forum discussions.

Already have an account? log in

shunhe on January 8, 2020

Hi @sprozes,

Thanks for the question! So we need a rule that we can substitute for the one that tells us L and O don't lecture n the same day as each other. Let me reproduce the set up here, which will help us visualize a little later:

Th: __ __ __
Fri: __ __ __
1 2 3

So there are three total slots for each day, and we know that M and N are on the same day (I'll just write them as MN now). But since L and O can't be on the same day, this means that MN can't be with J or K. Why? Because if MN is with J or K, then we force L and O to be together, since each day only has three slots and there are only two days. So we know that MN has to be with L or O, and J or K both have to be on the day that MN aren't on. We need another rule that basically expresses this relationship, and this is what (A) tells us. JK have to be on one day, and that means MN have to be on the other day, with O/L being split between the two. Hope this helps!